• 검색 결과가 없습니다.

Chapter Nine—Financial Statement Implications of

9. (b) is the correct answer. If money market rates average four percent during the year, the additional annual income (loss) from using the lockbox service would be $(6,000).

Increased daily cash receipt = $150,000 x 4 days

= $600,000

Interest earned on increase = $600,000 x .04 = $24,000 Annual fees = $2,500 X 12 months = $30,000

Net loss = + $24,000 - $30,000 = ($6,000)

10. (b) is the correct answer. Working capital is equal to current assets – current liabilities. The refinancing of

$50,000 of short term debt with long term debt decreases current liabilities and increases long term debt. The decrease in short term debt decreases current liabilities causing working capital to go up. Answer (a) is incorrect because the prepayment of $50,000 of next year's rent increases and decreases current assets, thus causing no change in working capital. Answer (c) is incorrect because the acquisition of land through the issuance of common stock has no effect on current assets or current liabilities, therefore no change in working capital. Answer (d) is incorrect because the purchase of temporary investments for cash increases and decreases current assets, thereby not changing working capital

11. (b) is the correct answer. The calculation of working capital is current assets - current liabilities. The current assets in this question are cash, accounts receivable, inventory and marketable securities. The current liabilities are accounts payable and mortgage payable - current. Under the current situation, the total current assets equal

$1,080,000 and total current liabilities equal $525,000 which calculates working capital to be $555,000. Under the proposal the current assets equal $1,280,000 and the current liabilities equal $755,000, calculating a working capital equal to $525,000. Given this, the effect of the plant expansion on Shaw's working capital is a decrease of $30,000 ($555,000 - $525,000).

12. (a) is the correct answer. Net working capital equals current assets minus current liabilities.

13. (d) is the correct answer. If the credit terms are 2/15 net 45, then a company not taking the 2% discount would have an extra 30 days (45 – 15) to use the money. Thirty days divided into a 360 day year equals twelve 30 day periods in a year. Therefore, if it costs 2% to hold the money an additional 30 days, and there are twelve 30 day periods in a year, then the approximate cost/benefit of the trade credit terms is 24% (2% x 12).

14. (c) is the correct answer. The optimal level of inventory would not be affected by the current level of inventory.

No matter what current level of inventory a company has, whether they have too much or too little, has no effect on the optimal level of inventory that the organization should have.

15. (a) is the correct answer. The maximum amount of new short-term debt the firm can assume in order to finance inventory without defaulting is $200,000.

If current liabilities are $200,000 and the current ratio equals 2, then the current assets equal $400,000.

1 2 s liabilitie current

$200,000

assets current

$400,000

So, to find the maximum amount of borrowing, using short term debt, without falling below the 1.5 current ratio, the following equation is used:

X = new borrowings

1 5 . 1 X

$200,000 X

$400,000

X = $200,000

9S-2

16. (d) is the correct answer. Inspections are not inventory carrying costs.

17. (d) is the correct answer. To earn maximum returns on investment assets is not a valid reason for a business to hold cash and marketable securities.

18. (d) is the correct answer. An example of a carrying cost is obsolescence.

19. (d) is the correct answer.

360 x .03

(45 – 10) 1.00 – .03

10.286 x .0309 = .3181 = 31.81%

20. (a) is the correct answer. Refinancing a short-term note payable with a two-year note payable would increase the working capital of a firm.

Current Assets - Current Liabilities = Working Capital Assuming CA = $10 and CL = $6, WC = $4. ($10 - $6 = $4).

If a company refinances $2 worth of short-term notes payable with a two year note payable, working capital would increase. ($10 - $4 = $6).

Answer (b) is incorrect. The purchase of a new plant financed by a 20-year mortgage would not increase the working capital of a firm. WC would stay the same. Answer (c) is incorrect. A cash collection of accounts receivable would not increase the working capital of a firm. WC would stay the same. Answer (d) is incorrect. A payment of a 20-year mortgage payable with cash would not increase the working capital of a firm. WC would decrease.

21. (d) is the correct answer. A decrease in carrying costs would increase the EOQ. A decrease in the denominator (carrying cost) would cause an increase in the EOQ based on the following formula:

unit per cost carrying

cost) (order (demand EOQ 2

The

Answer (a) is incorrect. A decrease in annual sales would not increase the EOQ. A decrease in demand would cause the EOQ to decrease. Answer (b) is incorrect. A decrease in cost per order would not increase the EOQ. A decrease in cost per order would cause the EOQ to decrease. Answer (c) is incorrect. A decrease in safety stock level would not increase the EOQ. A decrease in safety stock level would cause the EOQ to decrease.

22. (d) is the correct answer. When the Economic Order Quantity (EOQ) model is used for a firm which manufactures its inventory, ordering costs consist primarily of production set-up.

23. (d) is the correct answer. The cost of not taking a cash discount is generally higher than the cost of a bank loan.

This is intentional by the selling company to encourage early payment by customers. Answer (a) is incorrect. The cost of not taking a 2/10, net 30 cash discount is usually more than the prime rate. The annual cost of not taking the discount is approximately 37%.

02 x . cost 360

discount The

Answer (b) is incorrect. With trade terms of 2/15, net 60, if the discount is not taken, the buyer receives 45 days of free credit is not correct. The cost of the 45 days of credit is the 2%. Answer (c) is incorrect. The cost of not taking the discount is lower for terms of 2/10, net 60 than for 2/10, net 30. 2% for 50 days (60-10) is lower than paying 2%

for only 20 days (30-10) of using the money.

24. (c) is the correct answer. The expected cash inflow for Cooper in March is $119,000.

50% of credit sales for March = $ 90 X 50% = $ 45 30% of credit sales for February = $120 X 30% = $ 36 20% of credit sales for January = $100 X 20% = $ 20

Cash sales for March = U$ 18U

Total inflows for March (in thousands) = $119

25. (b) is the correct answer. The average inventory of Product A is 150 units.

Safety stock (50) + incoming order (200) = 250 units in inventory.

Inventory drops to 50 right before the next order is received.

Therefore average inventory = (250 + 50) / 2 = 150 Another way of coming up with the answer is:

2 order incoming (50)

stock safety inventory Average

26. (c) is the correct answer. No, Foster should not use the lock-box system, producing a loss of $20,000 per year.

Daily sales = $90 million / 360 = $250,000

3 days reduction in collection time = $250,000 X 3 = $750,000 available for investment At 8%, $750,000 will earn $60,000 annually

Earned $60,000 - lockbox cost of $80,000 = ($20,000)

A $20,000 loss would be incurred if Foster implements the lockbox system. Therefore do NOT use the lockbox system.

27. (c) is the correct answer as a conservative working capital policy results in an increase in working capital.

Therefore, current assets increase. Working capital = current assets minus current liabilities. A conservative working capital policy reduces the liquidity risk. Choice (a) is incorrect since a decrease in acid test ratio means that quick assets such as cash, receivables and marketable securities are decreasing in proportion to current liabilities and this results in lower working capital. Choice (b) is incorrect because increases in the ratio of current liabilities to noncurrent liabilities increases liquidity risk, and represents a less conservative working capital policy. Choice (d) is incorrect because as a company becomes more conservative in its working capital policy, it will not increase the funds invested in common stock (long term) and decrease funds invested in marketable securities (short term).

28. (b) is the correct answer since a draft delays the outflow of cash. A draft (check) is a 3-party instrument whereby the drawer orders the drawee to pay a certain sum of money to the payee. Choice (a) is incorrect since concentration banking does not delay the outflow of cash but instead regional banks may become centers for transfer of lockbox receipts. Choice (c) is incorrect since electronic data interchange does not delay the outflow of cash. Choice (d) is incorrect since a lockbox system is used to expedite the receipt of cash.

9S-4

29. (d) is the correct answer. If a firm increases its cash balance by issuing additional shares of common stock, working capital increases and the current ratio increases. Answer (a) is incorrect. If a firm increases its cash balance by issuing additional shares of common stock, working capital does not remain unchanged and the current ratio does not remain unchanged. Answer (b) is incorrect. If a firm increases its cash balance by issuing additional shares of common stock, working capital does increase but the current ratio does not remain unchanged. Answer (c) is incorrect. If a firm increases its cash balance by issuing additional shares of common stock, working capital does increase but the current ratio does not decrease.

30. (b) is the correct answer. The amount of inventory that a company would tend to hold in stock will increase as the cost of carrying inventory decreases. The larger the investment a firm makes in inventory, the higher the carrying costs. But if carrying costs decrease, the amount of inventory held in stock increases. Answer (a) is incorrect because if sales fall to a permanently low level, the amount of inventory held is adjusted downward accordingly. Answer (c) is incorrect since a company will tend to decrease the amount of inventory in stock as the variability in sales decreases. Answer (d) is incorrect since the company will tend to decrease the amount of inventory in stock as the cost of running out of stock decreases.

31. (a) A cash advance that is made to a divisional office does not change the current ratio or current assets. This is an intracompany transfer which doesn't change any accounts. Answer (b) is not correct. A cash dividend declaration increases current liabilities, thus changing the current ratio. Answer (c) is not correct. The retirement of short-term notes payable using cash reduces the current assets and reduces current liabilities, thus changing the current ratio:

3 2

6 2.5

1 – 3

1 – 6

Answer (d) is not correct. When a fully depreciated asset is sold for cash, cash increases which increases the current assets which in turn increases the current ratio.

32. (c) The formula for days sales outstanding (days of receivables) is as follows:

(40% x 15 days) + (.40 x 30 days) + (.20 x 45 days)

6 + 12 + 9 = 27

33. (d) The basic formula for calculating the cost of not taking the discount is as follows:

rate interest period x

discount days 360

This simple formula gives an answer that is a little less than the exact correct answer. Therefore, you will need to remember that the exact correct answer is larger than the answer obtained using the above formula. Therefore the answer is calculated as follows:

2%

10) x – (30

days 360

36%

2%

20 x days 360

So the exact answer is a little more than 36%: 36.7%.

34. (a) If vacant land is sold for less than the net book value, cash will go up which causes the current ratio to increase. In addition if the vacant land is sold for less than book value, there will be a loss causing a decrease in net income. Answer (b) is not correct. A long-term bond retired before maturity at a discount would generate a gain rather than a loss; therefore, the transaction would increase net income. Answer (c) is not correct. A stock dividend that is declared does not affect profit or the current ratio. The journal entry for a stock dividend declaration is as follows:

retained earnings x

common stock dividends distributable x

paid in capital x

Answer (d) is not correct. An uncollectible accounts receivable that is written off against the allowance account would not affect profit or the current ratio. The journal entry that would be made is as follows:

allowance for doubtful accounts x

accounts receivable x

35. (b) is the correct answer since at the end of the purchaser's operating cycle, he will get the necessary cash from the sale to pay off the supplier. Therefore, if the supplier extends credit for a longer period than the purchaser's operating cycle, he is in effect financing more than the purchaser's inventory needs. Choice (a) is incorrect since the seller will have a higher accounts receivable if the seller extends credit for a longer time period. Choice (c) is incorrect since the seller cannot be sure that the purchaser will convert the inventory to cash before payment is due.

Choice (d) is incorrect since the seller is not necessarily financing the purchaser's long-term assets, although the seller is financing more than the purchaser's inventory.

36. (a) All costs associated with the inventory after it arrives at our location are considered a carrying cost associated with the inventory. Shipping costs are not part of the carrying costs of inventory. The other choices are not correct.

All costs associated with the inventory after it arrives at our location are considered a carrying cost associated with the inventory.

37. (c) A compensating balance means that the $1,750,000 must stay in an account and not be used. The exact cost of this alternative equals 1,750,000 x .07 = 122,500. Answer (a) is not correct. This is not the least expensive; try again. The cost of this alternative is equal to 700 checks per day x 360 days per year x .50 fee per check = 126,000.

Answer (b) is not correct. This is not the least expensive. The cost of this alternative is equal to 700 checks per day x 360 days per year x $1,800 average per check x .0003 = 136,080. Answer (d) is not correct. This is not the least expensive. The cost of this alternative is equal to (700 checks per day x 360 days per year x .35 fee per check) + .0001 (700 checks per day x 360 days x $1,800 average per check) = 133,560.

38. (c) Logic would say that if the conversion period is every 80 days and the year is defined as 360 days, the inventory turnover would be 360 days / 80 days, or 4.5 times. Technically, the formula for inventory turnover could be used:

Days supply in inventory =

Turnover Inventory

360

9S-6